Jump to content

dogsteven's Content

There have been 1000 items by dogsteven (Search limited from 30-05-2020)



Sort by                Order  

#567444 Topic tổng hợp một số bất đẳng thức trong kì thi MO các nước

Posted by dogsteven on 22-06-2015 - 16:01 in Bất đẳng thức - Cực trị

áp dụng cauchy có:

$(\sum \frac{1}{2a^2+bc})^2\leq (\sum (b^2+c^2+bc))(\sum \frac{1}{(b^2+c^2+bc)(2a^2+bc)})$

mà $(b^2+c^2+bc)(2a^2+bc)\geq (ab+bc+ac)^2=1\Leftrightarrow \sum \frac{1}{(b^2+c^2+bc)(2a^2+bc)}\leq 3$

$\sum (bc+b^2+c^2)\leq 3(a^2+b^2+c^2)$

$bđt\Leftrightarrow \sqrt{\sum a^2}\leq \sum a^2$(luôn đúng)

phần cm $\geq 2$ mk thấy hih như sai đề

Phần đầu ta sử dụng CYH: $\sum \dfrac{1}{2a^2+bc}\geqslant \dfrac{4(a+b+c)^2}{\sum (2a^2+bc)(b+c)^2}$

Ta chuyển về chứng minh $p^2+9pr\geqslant 4$. Đồng bậc ta chuyển về chứng minh $-q^2+q+9r\geqslant 0$ với $p=2$

Với $q\leqslant 1$ thì bất đẳng thức hiển nhiên đúng.

Với $8>\dfrac{4}{3}\geqslant q\geqslant 1$ thì $-q^2+q+9r\geqslant -q^2+q+8(q-1)=(8-q)(q-1)\geqslant 0$

Ta có điều phải chứng minh.




#567627 Topic tổng hợp một số bất đẳng thức trong kì thi MO các nước

Posted by dogsteven on 23-06-2015 - 14:17 in Bất đẳng thức - Cực trị

Sau đây là lời giải cho bài toán trên:

Không mất tính tổng quát ta giả sử a$\geq$b$\geq$c

Khi đó dễ thấy rằng:$a\geq b\geq c$ $\frac{1}{b^2+8bc+3c^2}\geq \frac{1}{c^2+8ca+3a^2}\geq \frac{1}{a^2+8ab+3b^2}$

Áp dụng bất đẳng thức Chebysev và bất đẳng thức AM-GM, ta có:

$\sum \frac{a}{b^2+4bc+3c^2}\geq \frac{1}{3}(a+b+c)(\sum \frac{1}{b^2+4bc+3c^2})\geq 3(a+b+c)(\frac{1}{4(a+b+c)^2})\geq \frac{1}{4}$ (đpcm)

Nên xem lại.




#567215 Topic tổng hợp một số bất đẳng thức trong kì thi MO các nước

Posted by dogsteven on 21-06-2015 - 07:03 in Bất đẳng thức - Cực trị

Bài này thực sự là đặc chưng cho phương pháp tiếp tuyến, ta giải như sau:

1) Áp dụng bất đẳng thức AM-GM và giả thuyết thì chúng ta sẽ có BĐT đã cho tương đương BĐT sau:

$\sum \frac{1}{a^2-6a+13}\leq \frac{3}{8}$

Thật vậy lưu ý bất đẳng thức phụ sau:

$\frac{1}{a^2-6a+13}\leq \frac{a+1}{16}$

tương tự như thế suy ra$\sum \frac{1}{a^2-6a+13}\leq \frac{a+b+c+3}{16}=\frac{3}{8}$

từ đó ta có đpcm

Giả sử $a\geqslant b\geqslant c$. Bất đẳng thức trên tương đương với: $\sum \dfrac{(a+b)^2}{(a+b)^2+\dfrac{2(a+b)^2}{a^2+b^2}}\geqslant \dfrac{3}{2}$

Áp dụng bất đẳng thức Cauchy-Schwarz: $VT\geqslant \sum \dfrac{4(a+b+c)^2}{\sum (a+b)^2+\sum \dfrac{2(a+b)^2}{a^2+b^2}}$

Do đó ta cần chứng minh: $\sum (a+b)^2+\sum \dfrac{2(a+b)^2}{a^2+b^2}\leqslant 24$

$\Leftrightarrow \dfrac{4}{3} (a+b+c)^2-\sum (a+b)^2+12-\sum \dfrac{2(a+b)^2}{a^2+b^2}\geqslant 0\Leftrightarrow \sum (a-b)^2\left(\dfrac{6}{a^2+b^2}-1\right)\geqslant 0$

Nếu $a^2+b^2\leqslant 6$ thì bất đẳng thức đúng. Nếu $a^2+b^2\geqslant  6$ thì:

$\sum \dfrac{1}{a^2+b^2+2}\leqslant \dfrac{1}{8}+\dfrac{1}{a^2+2}+\dfrac{1}{b^2+6} \leqslant \dfrac{1}{8}+\dfrac{1}{8-b^2}+\dfrac{1}{b^2+2}\leqslant \dfrac{1}{8}+\dfrac{5}{8}=\dfrac{3}{4}$




#566886 Topic tổng hợp một số bất đẳng thức trong kì thi MO các nước

Posted by dogsteven on 19-06-2015 - 16:36 in Bất đẳng thức - Cực trị

Bài 120(Balkan Shortlist): Cho a,b,c >0. CMR nếu a+b+c=3 thì

 

 $\sqrt{\frac{a+b}{c+ab}}+\sqrt{\frac{b+c}{a+bc}}+\sqrt{\frac{c+a}{b+ca}}\geq 3$

Ta sẽ chứng minh: $(a+b)(b+c)(c+a)\geqslant (a+bc)(b+ca)(c+ab)$

Áp dụng bất đẳng thức AM-GM: $\sqrt{(a+bc)(b+ca)}\leqslant \dfrac{(a+b)(c+1)}{2}$

Tương tự ta có: $(a+bc)(b+ca)(c+ab)\leqslant \dfrac{(a+b)(b+c)(c+a)(a+1)(b+1)(c+1)}{8}\leqslant (a+b)(b+c)(c+a)$

AM-GM toàn bộ ba số trên và áp dụng cái bất đẳng thức đa chứng minh là ra.




#566882 Topic tổng hợp một số bất đẳng thức trong kì thi MO các nước

Posted by dogsteven on 19-06-2015 - 16:25 in Bất đẳng thức - Cực trị

Xin trình bày lời giải bài số 119 như sau:

 

Không mất tính tổng quát ta giả sử rằng $a\geq b\geq c$

 

khi đó ta dễ thấy rằng: $a^2\geq b^2\geq c^2$ ; $\frac{1}{b^2+c^2+bc}\geq \frac{1}{c^2+ca+a^2}\geq \frac{1}{a^2+ab+b^2}$

 

Áp dụng bất đẳng thức Chebysev,và bất đẳng thức AM-GM ta có:

$\frac{a^2}{b^2+bc+c^2}+\frac{b^2}{c^2+ca+a^2}+\frac{c^2}{a^2+ab+b^2}\geq \frac{1}{3}(a^2+b^2+c^2)(\sum \frac{1}{b^2+bc+c^2})\geq 3(a^2+b^2+c^2)(\frac{1}{2(a^2+b^2+c^2)+ab+bc+ca})\geq 3(a^2+b^2+c^2)(\frac{1}{3(a^2+b^2+c^2)})=1$

 

Vậy là ta có đpcm. :ukliam2:  :ukliam2:  :ukliam2: 

Áp dụng bất đẳng thức Cauchy-Schwarz:

$\sum \dfrac{a^2}{b^2+c^2+bc}\geqslant \dfrac{(a^2+b^2+c^2)^2}{2(a^2b^2+b^2c^2+c^2a^2)+abc(ab+b+c)}$

Mặt khác $(a^2+b^2+c^2)^2\geqslant 3(a^2b^2+b^2c^2+c^2a^2)\geqslant 3abc(a+b+c)$ nên

$\dfrac{(a^2+b^2+c^2)^2}{2(a^2b^2+b^2c^2+c^2a^2)+abc(ab+b+c)}\geqslant \dfrac{(a^2+b^2+c^2)^2}{\dfrac{2}{3}(a^2+b^2+c^2)^2+\dfrac{1}{3}(a^2+b^2+c^2)^2}=1$




#567628 Topic tổng hợp một số bất đẳng thức trong kì thi MO các nước

Posted by dogsteven on 23-06-2015 - 14:20 in Bất đẳng thức - Cực trị

Không biết bài này có chưa, nếu có rồi thì anh Đức xóa dùm em

Bài 142(APMO 1996) Cho a,b,c là độ dài 3 cạnh tam giác. Chứng minh rằng:

$\sqrt{a+b-c}+\sqrt{b+c-}+\sqrt{c+a-b}\geq \sqrt{a}+\sqrt{b}+\sqrt{c}$

Đẳng thức xảy ra khi nào

Bị ngược dấu rồi em.

Không mất tính tổng quát, giả sử $a\geslant b\geqslant c$

Dễ thấy $a+b-c\geqslant c+a-b\geqslant b+c-a$

Mà $a+b-c\geqslant a$ và $a+b-c+c+a-b=2a\geqslant a+b$ và $a+b-c+c+a-b+b+c-a=a+b+c$ nên $(a+b-c, c+a-b, b+c-a)\succ (a,b,c)$

Áp dụng bất đẳng thức Karamata cho hàm lõm $f(x)=\sqrt{x}$ cho ta điều phải chứng minh.




#568130 Topic tổng hợp một số bất đẳng thức trong kì thi MO các nước

Posted by dogsteven on 25-06-2015 - 17:07 in Bất đẳng thức - Cực trị

Bài 152(IMO shortlist): Cho a,b,c>0. CMR: nếu a+b+c=3 thì

$\sum \frac{1}{a^2+a+1}$$\leq 1$ 

Bất đẳng thức ngược chiều. (Có thể kiểm chứng bằng HFT (tiếp tuyến))

Có thể điều kiện là $abc=1$




#569456 Topic tổng hợp một số bất đẳng thức trong kì thi MO các nước

Posted by dogsteven on 02-07-2015 - 15:23 in Bất đẳng thức - Cực trị

Bài 167(China TST): Cho a,b,c thuộc [0;1]. Tìm GTNN của biểu thức sau:

                              P=$\frac{1}{(b-1)^2+a^2}+\frac{1}{(b-1)^2+c^2}$

Đề có thể sai chăng? Hiển nhiên $P\geqslant 1$, đẳng thức khi $a-1=b=c-1=0$

TST không thể dễ đến vậy




#568507 Topic tổng hợp một số bất đẳng thức trong kì thi MO các nước

Posted by dogsteven on 27-06-2015 - 17:17 in Bất đẳng thức - Cực trị

 Giả sử $a\geq b\geq c$

 Ta có : 

    $+)~~ a^3-5a+2=(a-2)(a^2+2a-1)\leq 0$

    $+)~~b^3+5a-5ab-1=(b-1)(b^2+b+1-5a)\leq (b-1)(a^2-4a+1)\leq 0$

    $+)~~c^3+5ab-5abc+1=(c-1)(c^2+c+1-5ab)\leq (c-1)(a^2+a+1-5a)\leq 0$

    $\Rightarrow a^3+b^3+c^3\leq 5abc$

   Dấu "=" xảy ra khi có một biến bằng 2, hai biến bằng 1

Đặt $f(a,b,c)=a^3+b^3+c^3-5abc$. Khi đó giả sử $2\geqslant a\geqslant b\geqslant c\geqslant 1$ thì ta có dãy bất đẳng thức:

$f(a,b,c)\leqslant g(2,b,c)\leqslant f(2,b,1)\leqslant f(2,1,1)=0$




#568323 Topic tổng hợp một số bất đẳng thức trong kì thi MO các nước

Posted by dogsteven on 26-06-2015 - 16:39 in Bất đẳng thức - Cực trị

Bài 155:(Slovenia TST): Cho a,b,c>0. CMR: $a^2+b^2+c^2=3$ thì

$\sum \frac{a}{b+2}\leq 1$

Bất đẳng thức trên tương đương với: $ab^2+bc^2+ca^2-abc\leqslant 2$

Giả sử $a=\text{median}\{b,c\}$ thì $bc^2+ca^2-a(c^2+bc)=c(a-c)(a-b)\leqslant 0$ nên $ab^2+bc^2+ca^2-abc\leqslant a(b^2+c^2)=a(3-a^2)\leqslant \dfrac{[4+(a-1)^2]^2}{8}\leqslant 2$ do $0\leqslant a\leqslant \sqrt{3}$

Từ đó suy ra điều phải chứng minh. Đẳng thức xảy ra khi và chỉ khi $a=b=c=1$




#568248 Topic tổng hợp một số bất đẳng thức trong kì thi MO các nước

Posted by dogsteven on 26-06-2015 - 09:22 in Bất đẳng thức - Cực trị

Bài 154 (Nghệ an2009).Cho $a,b,c>0$ thỏa mãn $abc=1$. Tìm giá trị nhỏ nhất $H=(ab)^3+(bc)^3+(ca)^3-6(a+b+c)$

Giả sử $a=\text{max}\{a,b,c\}$ thì $f(a,b,c)-f(a,\sqrt{bc},\sqrt{bc})=(\sqrt{b}-\sqrt{c})^2\left[a^3(b+c+\sqrt{bc})^2-6\right]\geqslant (\sqrt{b}-\sqrt{c})^2(9a^2-6)\geqslant 0$

Do đó ta chỉ cần chứng minh bất đẳng thức khi $a\geqslant b=c=t$. Thay trực tiếp $a$ theo $t$ và khảo sát hàm số.




#566834 Topic tổng hợp một số bất đẳng thức trong kì thi MO các nước

Posted by dogsteven on 19-06-2015 - 12:34 in Bất đẳng thức - Cực trị

Bài toán này đã có khá nhiều cách giải trên diễn đàn

http://diendantoanho...-frac2b-frac3c/

http://diendantoanho...c3cgeq-frac152/

Thêm một cách mà mình vừa nghĩ ra  :icon10: 

Từ điểm rơi của bài toán là $(a,b,c)=\left ( \frac{1}{3};\frac{4}{5};\frac{3}{2} \right )$

$\Rightarrow 36a=15b=8c=12$

Do đó ta đặt: $\left\{\begin{matrix} 36a=x & & & \\ 15b=y & & & \\ 8c=z & & & \end{matrix}\right.$

Bài toán trở thành: 

Cho $x,y,z>0$ thõa mãn: $\frac{7xy}{180}+\frac{yz}{60}+\frac{zx}{36} \leq 12$.TÌm GTNN của:

$P=\frac{36}{x}+\frac{30}{y}+\frac{24}{z}$

Từ giả thiết ta có: 

$2160 \geq 7xy+3yz+5zx \geq 15\sqrt[15]{x^7y^7.y^3z^3.z^5x^5}$

$\Rightarrow x^6y^5z^4 \leq 12^5$            (Theo $AM-GM$)

Do đó: $P=6(\frac{6}{x}+\frac{5}{y}+\frac{4}{z})\geq 6.15\sqrt[15]{\frac{1}{x^6y^5z^4}} \geq 90.\sqrt[15]{\frac{1}{12^{15}}}=\frac{15}{2}$

$\Rightarrow GTNN_{P}=\frac{15}{2}$ khi $(a,b,c)=\left ( \frac{1}{3};\frac{4}{5};\frac{3}{2} \right )$

[spoiler] Đề thi vòng 16 Vio 9 năm nay cũng ra bài này [\spoiler]

Lời giải không quan trọng, tìm được điểm rơi mới là điều tất yếu để giải quyết bài toán.

Không biết có cái hệ nào gọn hơn không chứ AM-GM suy rộng và Lagrange đều ra cái hệ như thế này:

$6yz-1=6zx-4=4xy-7$ và $2x+8y+21z=12xyz$ với $ax=by=cz=1$

Tuy nhiên, vẫn thích cách thế trực tiếp hơn, vừa tự nhiên, vừa đỡ tốn sức tìm điểm rơi.




#566364 Topic tổng hợp một số bất đẳng thức trong kì thi MO các nước

Posted by dogsteven on 17-06-2015 - 09:46 in Bất đẳng thức - Cực trị

Bài xx (IMO 2006). Tìm hằng số $M$ nhỏ nhất sao cho  bất đẳng thức

\[\left | ab(a^2-b^2)+bc(b^2-c^2)+ca(c^2-a^2) \right | \leqslant M(a^2+b^2+c^2)^2,\]

luôn đúng với mọi số thực $a,\,b,\,c$ thay đổi tùy ý.

Đặt $p=a+b+c, q=ab+bc+ca, r=abc$. Không mất tính tổng quát, chuẩn hóa $p=1$. Khi đó bất đẳng thức trở thành:

$q^2-4q^3+2(9q-2)r-27r^2\leqslant M^2(1-2q)^4$

Cố định $q$, ta xét hàm số vế trái theo $r$, do đó hàm số này đạt cực đại tại $r=\dfrac{9q-2}{27}$, do đó ta cần phải có: $M^2\geqslant \dfrac{4(1-3q)^3}{27(1-2q)^4}=f(q)$

Để $M^2\geqslant f(q)$ với mọi $q$ khi và chỉ khi $M^2\geqslant \text{max}f(q)$

Ta có $f'(q)=\dfrac{4(1+6q)(1-3q)^2}{27(2q-1)^5}=0\Leftrightarrow q=\dfrac{1}{3}$ hoặc $q=\dfrac{-1}{6}$

Giờ thế giá trị ta được $M\geqslant \dfrac{9\sqrt{2}}{32}$




#564473 Topic tổng hợp một số bất đẳng thức trong kì thi MO các nước

Posted by dogsteven on 08-06-2015 - 20:18 in Bất đẳng thức - Cực trị

 

Bài 62:
Trường hợp 1:$min{\{a,b,c}\} \geq -\frac{3}{4} \Rightarrow a,b,c \geq -\frac{3}{4}$
Ta cần chứng minh $\frac{a}{a^2+1} \leq \frac{36a+3}{50}$ (do xài pp tiếp tuyến nên biết cần cm cái này)
$ \Leftrightarrow \frac{(4a+3)(3a-1)^2}{50(a^2+1)} \geq 0 $
Tương tự ta được: $ \sum\frac{a}{a^2+1} \leq \frac{9}{10}$
Trường hợp 2:$min{\{a,b,c}\} < -\frac{3}{4} $
Giả sử $ c=min{\{a,b,c}\} \Rightarrow c<\frac{3}{4} $
Nếu $ a \geq 0 , b \leq 0 $ thì  $\frac{b}{b^2+1}+\frac{c}{c^2+1} <0$ và $\frac{a}{a^2+1} \leq \frac{1}{2}$ nên $VT < \frac{1}{2} < \frac{9}{10}$
Tương tự với $a \leq 0;b \geq 0$
Nếu $a \leq 0;b \leq 0$ thì $VT<0 < \frac{9}{10} $
Nếu $a \geq 0;b \geq 0$ thì ta có:

  1. Nếu $a \in [0;\frac{1}{2}] \cup [2;+\infty)$ thì $ \frac{a}{a^2+1} \leq \frac{2}{5} $.Kết hợp với $\frac{b}{b^2+1} \leq \frac{1}{2} ; \frac{c}{c^2+1} <0$ ta được $VT < \frac{9}{10}$
  2. Nếu $a \in [\frac{1}{2};2] $.
    -Xét $b \in [0;\frac{1}{2}] \cup [2;+\infty)$ thì  $ \frac{b}{b^2+1} \leq \frac{2}{5} $
    Kết hợp với $\frac{a}{a^2+1} \leq \frac{1}{2} ; \frac{c}{c^2+1} <0$ ta được $VT < \frac{9}{10}$
    -Xét $b \in [\frac{1}{2};2]$ thì ta có:$c=1-a-b \geq -3$
    Do $c \in [-3;-\frac{3}{4})$ nên $\frac{c}{c^2+1} < -\frac{1}{10}$
    Do đó ta có:$VT<\frac{1}{2}+\frac{1}{2}-\frac{1}{10}=\frac{9}{10} $.
    Vậy $\sum\frac{a}{a^2+1} \leq \frac{9}{10} $.Dấu "=" xảy ra khi $a=b=c=\frac{1}{3}$

 

Bất đẳng thức tương đương với: $\sum \dfrac{(a-1)^2}{a^2+1}\geqslant \dfrac{6}{5}$

Giả sử $(3b-1)(3c-1)\geqslant 0\Leftrightarrow b^2+c^2\leqslant \dfrac{1}{9}+\left(\dfrac{2}{3}-a\right)^2$

Đến đây ta có $\dfrac{(b-1)^2}{b^2+1}+\dfrac{(c-1)^2}{c^2+1}\geqslant \dfrac{9(a+1)^2}{19+\left(2-3a\right)^2}$

Đến đây biến đổi tương đương.




#565243 Topic tổng hợp một số bất đẳng thức trong kì thi MO các nước

Posted by dogsteven on 12-06-2015 - 18:19 in Bất đẳng thức - Cực trị

Em xin đóng góp một bài và rồi off game :))

Bài 104. Cho các số thực dương $a,b,c$. Chứng minh rằng: $\dfrac{a}{b}+\dfrac{b}{c}+\dfrac{c}{a}\geqslant 3\sqrt[4]{\dfrac{a^3+b^3+c^3}{3abc}}$




#564471 Topic tổng hợp một số bất đẳng thức trong kì thi MO các nước

Posted by dogsteven on 08-06-2015 - 20:11 in Bất đẳng thức - Cực trị

Bài 59:(PP điểm rơi trong AM-GM)(VNTST 2001)

Cho các số thực dương $a,b,c$ thỏa mãn: $2x+4y+7z=2xyz$. Tìm giá trị nhỏ nhất của biểu thức: $P=x+y+z$

 

p/s: trình bày luôn cách tìm điểm rơi nhé!

Ta dùng nhân tử để tìm điểm rơi.

Xét hàm số $L=x+y+z+\lambda(2x+4y+7z-2xyz)$

Đạo hàm riêng cho biến $x$ được $1=2\lambda (yz-1)$

Đạo hàm riêng cho biến $y$ được $1=2\lambda(zx-2)$

Đạo hàm riêng cho biến $z$ được $1=\lambda(2xy-7)$

Do đó $2(yz-1)=2(zx-2)=2xy-7$




#564469 Topic tổng hợp một số bất đẳng thức trong kì thi MO các nước

Posted by dogsteven on 08-06-2015 - 20:07 in Bất đẳng thức - Cực trị

 Bất đẳng thức đồng bậc 3 nên ta chuẩn hoá $a^2+b^2+c^2=9$

 Khi đó : BĐT $\Leftrightarrow 2(a+b+c)-abc\leq 10\Leftrightarrow a(2-bc)+(b+c).2\leq 10$

 Giả sử $a\geq b\geq c \Rightarrow |bc|\leq \frac{9-a^2}{2}\leq 3 \Rightarrow bc\in [-3;3]$

 Áp dụng Cauchy-Schwarz ta có : $a(2-bc)+(b+c).2\leq \sqrt{(9+2bc)(b^2c^2-4bc+8)}\leq 10\Leftrightarrow (bc+2)^2(2bc-7)\leq 0$

 Luôn đúng $\forall ~bc\in [-3;3]$

 Từ đó có đpcm

Ta có một lời giải rất xấu bằng phân tách trường hợp như sau:

Chuẩn hóa $a^2+b^2+c^2=9$. Giả sử $a\geqslant b\geqslant c$

Nếu $a\leqslant 0$ thì bất đẳng thức hiển nhiên đúng do $abc>-10$

Nếu $a\geqslant 0$ và $b,c\leqslant 0$ thì $2(a+b+c)\leqslant 2a\leqslant 6\leqslant 10\leqslant 10+abc$

Nếu $a\geqslant b\geqslant 0\geqslant c$ thì $2a+2b+2c\le 9+3c$ nên ta cần chứng minh $3c\le 1+\dfrac{c(9-c^2)}{2}$ luôn đúng.

Nếu $c\geqslant 0$ thì $2a+2b+2c\leqslant 9+c$

- Trường hợp $c\leqslant 1$ thì $9+c\leqslant 10\leqslant 10+abc$

- Trường hợp $c\geqslant 1$ thì $9+c\leqslant 9+abc\leqslant 10+abc$




#560970 Topic tổng hợp một số bất đẳng thức trong kì thi MO các nước

Posted by dogsteven on 22-05-2015 - 19:54 in Bất đẳng thức - Cực trị

Bài 41. 

Giả sử $x=\text{min}\{x,y,z\}$, Thay trực tiếp $x=\dfrac{4-yz}{y+z+yz}$ ta được:

$VT-VP=y^2+z^2+\dfrac{(4-yz)^2}{y+z+yz}-2\left(1+\sqrt{2}\right)\left(y+z+\dfrac{4-yz}{y+z+yz}\right)+3+6\sqrt{2}$

Đặt $a=y+z$ và $b=yz$ thì ta có $f(b)=a^2-2b+\dfrac{(4-b)^2}{(a+b)^2}-2(1+\sqrt{2})\left(a+\dfrac{4-b}{a+b}\right)+3+6\sqrt{2}$

$=-2b+\dfrac{4-b}{(a+b)^2}\left[4-b-2(1+\sqrt{2})(a+b)\right]+a^2-2(1+\sqrt{2})a+3+6\sqrt{2}$

Có:

$4-b-2(1+\sqrt{2})(a+b)\geqslant 4-\dfrac{a^2}{4}-2(1+\sqrt{2})\left(a+\dfrac{a^2}{4}\right)$

$4-b\geqslant 4-\dfrac{a^2}{4}$

$-2b\geqslant \dfrac{a^2}{2}$

$\dfrac{1}{(a+b)^2}\geqslant \dfrac{1}{\left(a+\dfrac{a^2}{4}\right)^2}$

Do đó ta chỉ cần chứng minh khi $b=\dfrac{a^2}{4}$




#565271 Topic tổng hợp một số bất đẳng thức trong kì thi MO các nước

Posted by dogsteven on 12-06-2015 - 20:14 in Bất đẳng thức - Cực trị

Spoiler

Bổ đề:

 

$(x+y+z)^3\geq \frac{4}{27}(x^2y+y^2z+z^2x+xyz)$

 

Trong đó lấy $x=\frac{a}{b},y=\frac{b}{c},z=\frac{c}{a}$ thì ta có:

 

$(\frac{a}{b}+\frac{b}{c}+\frac{c}{a})^3\geq \frac{27}{4}(\frac{a^3+b^3+c^3}{abc}+1)$

 

Do đó ta chỉ cần chứng minh:

 

$\frac{27}{4}(\frac{a^3+b^3+c^3}{abc}+1)\geq 3^3.(\frac{a^3+b^3+c^3}{3abc})^{\frac{3}{4}}$

 

Cái này đặt ẩn rồi biến đổi tương đương được BĐT đúng. :lol:

 

Luôn đúng theo AM-GM 4 số: $3.\dfrac{a^3+b^3+c^3}{3abc}+1$ :D




#565669 Topic tổng hợp một số bất đẳng thức trong kì thi MO các nước

Posted by dogsteven on 14-06-2015 - 12:37 in Bất đẳng thức - Cực trị

Bài 105: (Japanese 1997) Cho $a,b,c$ là những số thực dương.Chứng minh rằng $\frac{(b+c-a)^{2}}{a^{2}+(b+c)^{2}}+\frac{(a+c-b)^{2}}{b^{2}+(a+c)^{2}}+\frac{(b+a-c)^{2}}{c^{2}+(b+a)^{2}}\geq \frac{3}{5}$

Bất đẳng thức trên tương đương với: $\sum \dfrac{a(b+c)}{a^2+(b+c)^2}\leqslant \dfrac{6}{5}$

Áp dụng bất đẳng thức Cauchy-Schwarz: $a^2+(b+c)^2\geqslant a(b+c)+\dfrac{3(b+c)^2}{4}$

Do đó ta có $\sum \dfrac{a(b+c)}{a^2+(b+c)^2}\leqslant \sum \dfrac{4a(b+c)}{4a(b+c)+3(b+c)^2}=3-\sum \dfrac{3(b+c)^2}{4a(b+c)+3(b+c)^2}$

Áp dụng bất đẳng thức Cauchy-Schwarz: $\sum \dfrac{(b+c)^2}{4a(b+c)+3(b+c)^2}\geqslant \dfrac{2(a+b+c)^2}{3(a^2+b^2+c^2)+7(ab+bc+ca)}\geqslant \dfrac{3}{5}$

Do đó $3-\sum \dfrac{3(b+c)^2}{4a(b+c)+3(b+c)^2}\leqslant \dfrac{6}{5}$, đây là điều ta cần chứng minh.




#565871 Topic tổng hợp một số bất đẳng thức trong kì thi MO các nước

Posted by dogsteven on 15-06-2015 - 10:24 in Bất đẳng thức - Cực trị

spam : Học được mấy cái hay ho này ở đâu thế. xin cái bí kíp 

Mua sách anh ơi, em cũng có mấy tài liệu lập trình chuyên môn nữa, của ông anh :))




#565761 Topic tổng hợp một số bất đẳng thức trong kì thi MO các nước

Posted by dogsteven on 14-06-2015 - 21:21 in Bất đẳng thức - Cực trị

chú em dogsteven học với tốc độ thế này thì quá tốt,hy vọng được nghe tin tốt lành từ chú em ở VMO 2 năm nữa :))

( hơi spam )

Em định không thi VMO, mấy năm học chuyên em học lập trình game với photoshop :)) (đam mê)

(hơi spam)




#565758 Topic tổng hợp một số bất đẳng thức trong kì thi MO các nước

Posted by dogsteven on 14-06-2015 - 21:17 in Bất đẳng thức - Cực trị

Giờ đã học hết cả chương trình PT thế này rồi à em. Anh giờ còn chưa biết mấy cái $In$ này là gì :unsure:

Em học cho vui.

P.s. Đây có phải spam không nhỉ :))




#565749 Topic tổng hợp một số bất đẳng thức trong kì thi MO các nước

Posted by dogsteven on 14-06-2015 - 21:04 in Bất đẳng thức - Cực trị

Tổng quát cách làm:

Với bất đẳng thức có dạng tổng hàm $f(x)+f(y)+f(z)\geqslant 0$ với điều kiện $x+y+z=3$ ta có thể sẽ chứng minh: $f(x)\geqslant f'(1)(x-1)+f(1)$

Với bất đẳng thức có dạng tổng hàm $f(x)+f(y)+f(z)\geqslant 0$ với điều kiện $xyz=1$ thì ta có thể sẽ chứng minh: $f(x)\geqslant f'(x)\ln x+f(1)$




#559479 Topic tổng hợp một số bất đẳng thức trong kì thi MO các nước

Posted by dogsteven on 15-05-2015 - 10:04 in Bất đẳng thức - Cực trị

Bài 6. $\sum a . \sum \dfrac{1}{a} - 9 =\sum \dfrac{(b-c)^2}{bc}\geqslant 3\sqrt[3]{\dfrac{\prod (b-c)^2}{(abc)^2}}$